Sets of Consecutive Positive Integers With Sum k (2006 AMC 12A #8)

2006 AMC 12A #8

Problem: How many sets of two or more consecutive positive integers have a sum of 15?

Solution: It is simple to see there are 3 total ways: {1, 2, 3, 4, 5} | {4, 5, 6} | {7, 8}.

But why? Is there a way to generalize this?

Yes! The number of sets of two or more positive integers that have a sum of k is equal to the number of positive odd divisors of k excluding 1.

Let's break this down with an example. Take 369 = 3^2 * 41. From above, we have 5 ways.

To find such sets, we take the factors of 369 and let them be our median of the set. Then, (regardless if any of the integers in the set are negative) we have a feasible set.

First, 123 will be our median. Our set is: {122, 123, 124}. This set has 3 numbers and needs no manipulation.

Next, our median equals 41. Our set becomes longer: {37, 38, 39, 40, 41, 42, 43, 44, 45}. This also doesn't require manipulation.

Now, we have 9 as our median. Our set is then: {-11, -10 , -9, ... 9, 10, 11, ... 29}. However, this set has negative numbers which we don't want. So after canceling the positive and negative numbers, we get: {12, 13, 14, ..., 27, 28, 29} which has 18 = 2*9 values.

Let's take 3 as the median. Our set is: {-58, ..., 3, ..., 64}. But as we can see, the negative numbers cancel out the positive ones to leave us with the set: {59, 60, 61, 62, 63, 64}, which add up to 369. Note that this set has 6 = 2*3 numbers.

Lastly, we have 1 to be a median. This is the set: {-183, ..., 1, ... 185} --> {184, 185} which has 2 = 2*1 numbers.

Now we see that there are indeed 5 ways to express 369 as the sum of 2 or more consecutive positive integers. (369 cannot be a median or else there will only be 1 integer in the set.) But how can we be sure?

Well, we first let's look at how we chose our medians. Our medians are all the factors of 369, or x, such that 369/x is an odd integer. This ensures that the sequence of 369/x integers with a median of x will indeed sum to 369 and that all numbers in the sequence are integers. So the x we chose were: 1, 3, 9, 41, 123, (no 369).

When x = 1, 3, 9 (or when 369/(2x) > x), our sequence went into the negatives. This cancelled out the positives to give us 2x numbers in all. But why 2x? Suppose 369/x = y. Then our sequence went from x - (y-1)/2 to x + (y-1)/2 for a total of x + (y-1)/2 - (-(x - (y-1)/2)) = 2x.

On the other hand, when x = 41, 123 (or when 369/2x < x), our sequence did not go into the negatives. This resulted in 369/x numbers with x as the median.

But how do we know we didn't lose any possibilities?

The only other ways to get a possible set is 369/(2n), which gives us n/2 as the median. However, our  369/(2x) > x cases covered those possibilities with 2x numbers total with n = x.

Comments

Popular posts from this blog

Math Formulas + Facts

Discovered New Theorem while solving Chinese Remainder Theorem

Cyclic Quattrocento (Fall 2021 AMC 12B #24)